Difference between revisions of "2021 Fall AMC 10B Problems/Problem 5"

(Tag: New redirect)
 
Line 1: Line 1:
==Problem 5==
+
#REDIRECT [[2021_Fall_AMC_12B_Problems/Problem_4]]
Let <math>n=8^{2022}</math>. Which of the following is equal to <math>\frac{n}{4}?</math>
 
 
 
<math>(\textbf{A})\: 4^{1010}\qquad(\textbf{B}) \: 2^{2022}\qquad(\textbf{C}) \: 8^{2018}\qquad(\textbf{D}) \: 4^{3031}\qquad(\textbf{E}) \: 4^{3032}</math>
 
 
 
==Solution 1==
 
We have <cmath>n=8^{2022}=  \left(8^\frac{2}{3}\right)^{2022}=4^{3033}.</cmath>
 
Therefore, <cmath>\frac{n}4=\boxed{(\textbf{E})\:4^{3032}}.</cmath>
 
 
 
~kingofpineapplz
 
 
 
 
 
==See Also==
 
{{AMC10 box|year=2021 Fall|ab=B|num-a=6|num-b=4}}
 
{{MAA Notice}}
 

Latest revision as of 01:17, 24 November 2021